Let’s test Jow Forums Hard mode

Ok the non brainlets got the first one.

Now this one,

50 LINK For the first correct solution.

Leave an address

Attached: E8B9C830-287C-435B-BF74-66B69DB8CE29.jpg (1125x986, 249K)

Other urls found in this thread:

mathwords.com/a/arc_length_of_a_curve.htm
physics.stackexchange.com/questions/288824/maximum-trajectory-length
wolframalpha.com/input/?i=integrate sqrt((cos a)^2 + t^2*g^2+2*t*g*(cos(0.5*pi-a))+(cos(0.5pi-a))^2) dt from 0 to (2/g*cos(0.5pi-a))
wolframalpha.com/input/?i=integrate sqrt((cos a)^2 +(cos(0.5pi-a)-t*g)^2) dt from 0 to (2/g*cos(0.5pi-a))
twitter.com/NSFWRedditVideo

>Doing this niggers homework

And no it’s not horizontal distance

45 degrees for that perfect sin curve.

See

I aint doing calculus for you nigga

56 degrees

Straight up 90 degrees yo

Exact solution

>maximum distance in the air

Literally 90° if maximizing air distance is the goal, questions wording is retarded though doesn't specify which direction air distance is to be maximized

If you just wanna maximize air DISTANCE, ie direction doesn't matter then fire it straight up

how did you edit your post? trip?

Mod
>god

No, because then all the force is being used against gravity, remember we are neglecting air resistance

45 degrees. What a retard, everyone knows that since before high school.

Not horizontal distance
Flight path

alright then, 0.000000000000etc1 degrees

Fire 360°. Walk away.

180°

Kys.

>neglecting air resistance

nrm then, with nothing impeding the horizontal travelling force then yeah it'll be an angle less than 90° to maximize air distance

Wouldn't you also need the shape/weight/velocity of the projectile?

R=v0^2sin2θ0 /g

air resistance 35 deg
no air resistance: 45gef

>what are variables?
Fuck me, Jow Forums is retarded. How are you investing if you don't understand basic math?

56.4658

If I won then you can make a donation to the society of St Vincent de Paul.

Thanks

(the angle should be from the centre of the bore of the cannon to the horizontal)

You want to find what the maximum of x + y could be. If you fired it straight up, x would be 0

Actually I change my mind OP is interpreting the question wrong. Teacher just doesn't want you to include the projectile bouncing

This IS the angle that will give the longest trajectory.

Bonus points for latex?

I'm from a third world so pls no bully

0x3faa042d0e9fbeaef324f5b158831f8bc7351f24

Attached: linkies.png (560x416, 23K)

wow brainlet trying to shit on other brainlet
>Neglect air resistance

From where do you derive the first formula ?

damn yall are dumb fucking niggers couldn't even use google
mathwords.com/a/arc_length_of_a_curve.htm

It's not particularly basic to be honest with you. I'm a grad engineer and into ballistics so I could just use the correct terms to find the anger in a search. I don't thing its with in the math of the average person doing applied math at 18. Just a notch above. First year undergrad level

here

physics.stackexchange.com/questions/288824/maximum-trajectory-length

I bet OP having them cold sweats right

Attached: OPdrool.gif (240x180, 2M)

Well OP?
>56.4658

what the fuck are you talking about? Weight and shape of the flying object is irrelevant. I hope you were answering someone else, or I fear you will not make it.

Let me quote that again
>Neglect air resistance

LITERALLY MAKE A FUNCTION AND SOLVE IT BRAINLETTE

Just realised OP very clever. I hear and understand

I tried solving this unlike the other brainlets, and we end up having to solve an equation which inbolves both polynomial terms in x and terms with arcsinh(x).

There is probably no analytical solution to this equation, so fuck you OP

48

Just noticed this post, as you can see we cannot solve this problem analytically

That does neglect air resistance otherwise you would need to supply the ballistic coefficient of the projectile. This also assumes a smoothbore cannon

>t. brainlet nigger

I wrote a math report about this in high school. Forgot the formula though.. Probably straight up / 90 degrees.

>no gravitational acceleration

shouldn't it never touch the ground again once it's fired as long as it points upwards

Bout tree fidty

got as far to make wolfram alpha timeout
wolframalpha.com/input/?i=integrate sqrt((cos a)^2 + t^2*g^2+2*t*g*(cos(0.5*pi-a))+(cos(0.5pi-a))^2) dt from 0 to (2/g*cos(0.5pi-a))

>Just noticed this post, as you can see we cannot solve this problem analytically

Meh. I could ( readily I have a ballistic calc on my phone). The fact is that the question is a basic formulaic one with little real application as the canon is unrifled.unknown (we would need to know the twist rate and barrel length), there is no BC, we have no air density, temperature, altitude, if we are being accurate the powder burn temperature, the muzzle velocity, the weight of the projectile in grains and the projectile diameter the latitude and longitude, compass direction of muzzle, the height of the cannon chassis above ground etc etc .

Ask a textbook question and get a textbook answer, which I gave you correctly here

>56.4658

and since I am not a liar I highlighted that here.
Do did not tell me I had to solve it from first principles and the full formula to solve it is as cited.

oh nvm i read it wrong

>remember we are neglecting air resistance
Creatura physics

Attached: 1510844231478.png (500x470, 18K)

Just take a number of line integrals along different trajectories

bc is ballistic coefficient by the way....the mathematical unit representing the projectiles response to wind resistance. By the way OP I actually did a paper based on hollow centred projectiles and the gyroscopic effect in enhancing efficiency once apon a time. sorry I am such a brainlet to you.

I hold link by the way. I just thought it would be more Christian if you made a smaller donation to the Vincent de Paul instead

(You)
should be
wolframalpha.com/input/?i=integrate sqrt((cos a)^2 +(cos(0.5pi-a)-t*g)^2) dt from 0 to (2/g*cos(0.5pi-a))
negative sign was missing
this is simply integral (0, t_end) |velocity| dt
from I see this was the correct approach
(didn't want stinkies, just got curious)

Are bounces allowed?

>im a grad engineer so i googled it
>can't even into Lagrangians
typical engineers

Nobody else got it

Attached: Queen_Elizabeth_II_in_March_2015 (1).jpg (1698x2242, 1.01M)

The apex of the curvature of the Earth for any given distance is 8 inches x the distance in miles squared.

So if you wanted to hit something 3 miles away, the arc of the cannonball would have to be at least 6 feet.
The further the distance, the exponentially higher the arc.
So for example, if you wanted to hit something 40 miles away, the cannonball arc would have to be at least 1066 feet.


Now a navy railgun is a line of sight weapon, like a gun. It shoots in a straight line. You have to be able to see the target to shoot it.
It hits targets over a 100 miles away.
It shoots in a straight line to a target that should be obscured by a 6667 foot high apex.

Either railguns are make believe or the Earth is flat

Attached: IMG_20170124_210254.jpg (1564x1564, 636K)

Or maybe.. Light doesn't always travel in a perfectly straight line?